Group of order $6p^m$ is solvable for prime $pgeq 7$












2












$begingroup$



Let $pgeq 7$ be a prime and $m$ be a positive integer. Prove that group of order $6p^m$ is solvable.




Attempt:



By Sylow's theorems we have that $n_p mid 6$ so $n_pin {1,2,3,6}$ where $n_p$ is the number of Sylow $p$ groups. Also we have that $n_p equiv 1 pmod p$ so $n_p=1$ and is thus normal. So we investigate:



$$Gtrianglerighteq H_p$$



Where $H_p$ denotes the Sylow $p$ group. We know that $|G/H_p|=6$ and there are two groups of order $6$: $Bbb Z_6$ and $S_3$. Both are solvable. We also know that Sylow groups are solvable. Since $G/H_p$ is solvable and $H_p$ is solvable, $G$ is solvable.



Is this correct?










share|cite|improve this question











$endgroup$












  • $begingroup$
    Well, as you've noticed denoting as $;S_7;$ a Sylow subgroup is a very bad idea...so edit your question and change it!
    $endgroup$
    – DonAntonio
    Dec 7 '18 at 20:58










  • $begingroup$
    @DonAntonio there.
    $endgroup$
    – Zachary Selk
    Dec 7 '18 at 21:16










  • $begingroup$
    you are using $7$ but you really mean $p$
    $endgroup$
    – the_fox
    Dec 8 '18 at 3:52










  • $begingroup$
    @the_fox sorry, of course
    $endgroup$
    – Zachary Selk
    Dec 8 '18 at 4:09










  • $begingroup$
    See also this question, with similar arguments.
    $endgroup$
    – Dietrich Burde
    Dec 8 '18 at 9:03


















2












$begingroup$



Let $pgeq 7$ be a prime and $m$ be a positive integer. Prove that group of order $6p^m$ is solvable.




Attempt:



By Sylow's theorems we have that $n_p mid 6$ so $n_pin {1,2,3,6}$ where $n_p$ is the number of Sylow $p$ groups. Also we have that $n_p equiv 1 pmod p$ so $n_p=1$ and is thus normal. So we investigate:



$$Gtrianglerighteq H_p$$



Where $H_p$ denotes the Sylow $p$ group. We know that $|G/H_p|=6$ and there are two groups of order $6$: $Bbb Z_6$ and $S_3$. Both are solvable. We also know that Sylow groups are solvable. Since $G/H_p$ is solvable and $H_p$ is solvable, $G$ is solvable.



Is this correct?










share|cite|improve this question











$endgroup$












  • $begingroup$
    Well, as you've noticed denoting as $;S_7;$ a Sylow subgroup is a very bad idea...so edit your question and change it!
    $endgroup$
    – DonAntonio
    Dec 7 '18 at 20:58










  • $begingroup$
    @DonAntonio there.
    $endgroup$
    – Zachary Selk
    Dec 7 '18 at 21:16










  • $begingroup$
    you are using $7$ but you really mean $p$
    $endgroup$
    – the_fox
    Dec 8 '18 at 3:52










  • $begingroup$
    @the_fox sorry, of course
    $endgroup$
    – Zachary Selk
    Dec 8 '18 at 4:09










  • $begingroup$
    See also this question, with similar arguments.
    $endgroup$
    – Dietrich Burde
    Dec 8 '18 at 9:03
















2












2








2


0



$begingroup$



Let $pgeq 7$ be a prime and $m$ be a positive integer. Prove that group of order $6p^m$ is solvable.




Attempt:



By Sylow's theorems we have that $n_p mid 6$ so $n_pin {1,2,3,6}$ where $n_p$ is the number of Sylow $p$ groups. Also we have that $n_p equiv 1 pmod p$ so $n_p=1$ and is thus normal. So we investigate:



$$Gtrianglerighteq H_p$$



Where $H_p$ denotes the Sylow $p$ group. We know that $|G/H_p|=6$ and there are two groups of order $6$: $Bbb Z_6$ and $S_3$. Both are solvable. We also know that Sylow groups are solvable. Since $G/H_p$ is solvable and $H_p$ is solvable, $G$ is solvable.



Is this correct?










share|cite|improve this question











$endgroup$





Let $pgeq 7$ be a prime and $m$ be a positive integer. Prove that group of order $6p^m$ is solvable.




Attempt:



By Sylow's theorems we have that $n_p mid 6$ so $n_pin {1,2,3,6}$ where $n_p$ is the number of Sylow $p$ groups. Also we have that $n_p equiv 1 pmod p$ so $n_p=1$ and is thus normal. So we investigate:



$$Gtrianglerighteq H_p$$



Where $H_p$ denotes the Sylow $p$ group. We know that $|G/H_p|=6$ and there are two groups of order $6$: $Bbb Z_6$ and $S_3$. Both are solvable. We also know that Sylow groups are solvable. Since $G/H_p$ is solvable and $H_p$ is solvable, $G$ is solvable.



Is this correct?







abstract-algebra group-theory sylow-theory






share|cite|improve this question















share|cite|improve this question













share|cite|improve this question




share|cite|improve this question








edited Dec 8 '18 at 4:09







Zachary Selk

















asked Dec 7 '18 at 20:24









Zachary SelkZachary Selk

593311




593311












  • $begingroup$
    Well, as you've noticed denoting as $;S_7;$ a Sylow subgroup is a very bad idea...so edit your question and change it!
    $endgroup$
    – DonAntonio
    Dec 7 '18 at 20:58










  • $begingroup$
    @DonAntonio there.
    $endgroup$
    – Zachary Selk
    Dec 7 '18 at 21:16










  • $begingroup$
    you are using $7$ but you really mean $p$
    $endgroup$
    – the_fox
    Dec 8 '18 at 3:52










  • $begingroup$
    @the_fox sorry, of course
    $endgroup$
    – Zachary Selk
    Dec 8 '18 at 4:09










  • $begingroup$
    See also this question, with similar arguments.
    $endgroup$
    – Dietrich Burde
    Dec 8 '18 at 9:03




















  • $begingroup$
    Well, as you've noticed denoting as $;S_7;$ a Sylow subgroup is a very bad idea...so edit your question and change it!
    $endgroup$
    – DonAntonio
    Dec 7 '18 at 20:58










  • $begingroup$
    @DonAntonio there.
    $endgroup$
    – Zachary Selk
    Dec 7 '18 at 21:16










  • $begingroup$
    you are using $7$ but you really mean $p$
    $endgroup$
    – the_fox
    Dec 8 '18 at 3:52










  • $begingroup$
    @the_fox sorry, of course
    $endgroup$
    – Zachary Selk
    Dec 8 '18 at 4:09










  • $begingroup$
    See also this question, with similar arguments.
    $endgroup$
    – Dietrich Burde
    Dec 8 '18 at 9:03


















$begingroup$
Well, as you've noticed denoting as $;S_7;$ a Sylow subgroup is a very bad idea...so edit your question and change it!
$endgroup$
– DonAntonio
Dec 7 '18 at 20:58




$begingroup$
Well, as you've noticed denoting as $;S_7;$ a Sylow subgroup is a very bad idea...so edit your question and change it!
$endgroup$
– DonAntonio
Dec 7 '18 at 20:58












$begingroup$
@DonAntonio there.
$endgroup$
– Zachary Selk
Dec 7 '18 at 21:16




$begingroup$
@DonAntonio there.
$endgroup$
– Zachary Selk
Dec 7 '18 at 21:16












$begingroup$
you are using $7$ but you really mean $p$
$endgroup$
– the_fox
Dec 8 '18 at 3:52




$begingroup$
you are using $7$ but you really mean $p$
$endgroup$
– the_fox
Dec 8 '18 at 3:52












$begingroup$
@the_fox sorry, of course
$endgroup$
– Zachary Selk
Dec 8 '18 at 4:09




$begingroup$
@the_fox sorry, of course
$endgroup$
– Zachary Selk
Dec 8 '18 at 4:09












$begingroup$
See also this question, with similar arguments.
$endgroup$
– Dietrich Burde
Dec 8 '18 at 9:03






$begingroup$
See also this question, with similar arguments.
$endgroup$
– Dietrich Burde
Dec 8 '18 at 9:03












1 Answer
1






active

oldest

votes


















3












$begingroup$

By the way, there's no need to take $pge 7$. The result holds for all primes $p$. For $p=2,3$ this follows from Burnside's $p^aq^b$ theorem. For $p=5$ you additionally use the result that any group of order divisible by 2 but not 4 has an index 2 subgroup (of even permutations in the regular representation). And that proof also works for larger $p$ as well, and so is another solution to the original question






share|cite|improve this answer











$endgroup$













  • $begingroup$
    See here for the result about groups of order $equiv2pmod4$.
    $endgroup$
    – Jyrki Lahtonen
    Dec 30 '18 at 14:23











Your Answer





StackExchange.ifUsing("editor", function () {
return StackExchange.using("mathjaxEditing", function () {
StackExchange.MarkdownEditor.creationCallbacks.add(function (editor, postfix) {
StackExchange.mathjaxEditing.prepareWmdForMathJax(editor, postfix, [["$", "$"], ["\\(","\\)"]]);
});
});
}, "mathjax-editing");

StackExchange.ready(function() {
var channelOptions = {
tags: "".split(" "),
id: "69"
};
initTagRenderer("".split(" "), "".split(" "), channelOptions);

StackExchange.using("externalEditor", function() {
// Have to fire editor after snippets, if snippets enabled
if (StackExchange.settings.snippets.snippetsEnabled) {
StackExchange.using("snippets", function() {
createEditor();
});
}
else {
createEditor();
}
});

function createEditor() {
StackExchange.prepareEditor({
heartbeatType: 'answer',
autoActivateHeartbeat: false,
convertImagesToLinks: true,
noModals: true,
showLowRepImageUploadWarning: true,
reputationToPostImages: 10,
bindNavPrevention: true,
postfix: "",
imageUploader: {
brandingHtml: "Powered by u003ca class="icon-imgur-white" href="https://imgur.com/"u003eu003c/au003e",
contentPolicyHtml: "User contributions licensed under u003ca href="https://creativecommons.org/licenses/by-sa/3.0/"u003ecc by-sa 3.0 with attribution requiredu003c/au003e u003ca href="https://stackoverflow.com/legal/content-policy"u003e(content policy)u003c/au003e",
allowUrls: true
},
noCode: true, onDemand: true,
discardSelector: ".discard-answer"
,immediatelyShowMarkdownHelp:true
});


}
});














draft saved

draft discarded


















StackExchange.ready(
function () {
StackExchange.openid.initPostLogin('.new-post-login', 'https%3a%2f%2fmath.stackexchange.com%2fquestions%2f3030334%2fgroup-of-order-6pm-is-solvable-for-prime-p-geq-7%23new-answer', 'question_page');
}
);

Post as a guest















Required, but never shown

























1 Answer
1






active

oldest

votes








1 Answer
1






active

oldest

votes









active

oldest

votes






active

oldest

votes









3












$begingroup$

By the way, there's no need to take $pge 7$. The result holds for all primes $p$. For $p=2,3$ this follows from Burnside's $p^aq^b$ theorem. For $p=5$ you additionally use the result that any group of order divisible by 2 but not 4 has an index 2 subgroup (of even permutations in the regular representation). And that proof also works for larger $p$ as well, and so is another solution to the original question






share|cite|improve this answer











$endgroup$













  • $begingroup$
    See here for the result about groups of order $equiv2pmod4$.
    $endgroup$
    – Jyrki Lahtonen
    Dec 30 '18 at 14:23
















3












$begingroup$

By the way, there's no need to take $pge 7$. The result holds for all primes $p$. For $p=2,3$ this follows from Burnside's $p^aq^b$ theorem. For $p=5$ you additionally use the result that any group of order divisible by 2 but not 4 has an index 2 subgroup (of even permutations in the regular representation). And that proof also works for larger $p$ as well, and so is another solution to the original question






share|cite|improve this answer











$endgroup$













  • $begingroup$
    See here for the result about groups of order $equiv2pmod4$.
    $endgroup$
    – Jyrki Lahtonen
    Dec 30 '18 at 14:23














3












3








3





$begingroup$

By the way, there's no need to take $pge 7$. The result holds for all primes $p$. For $p=2,3$ this follows from Burnside's $p^aq^b$ theorem. For $p=5$ you additionally use the result that any group of order divisible by 2 but not 4 has an index 2 subgroup (of even permutations in the regular representation). And that proof also works for larger $p$ as well, and so is another solution to the original question






share|cite|improve this answer











$endgroup$



By the way, there's no need to take $pge 7$. The result holds for all primes $p$. For $p=2,3$ this follows from Burnside's $p^aq^b$ theorem. For $p=5$ you additionally use the result that any group of order divisible by 2 but not 4 has an index 2 subgroup (of even permutations in the regular representation). And that proof also works for larger $p$ as well, and so is another solution to the original question







share|cite|improve this answer














share|cite|improve this answer



share|cite|improve this answer








edited Dec 7 '18 at 22:57

























answered Dec 7 '18 at 22:49









C MonsourC Monsour

6,2191325




6,2191325












  • $begingroup$
    See here for the result about groups of order $equiv2pmod4$.
    $endgroup$
    – Jyrki Lahtonen
    Dec 30 '18 at 14:23


















  • $begingroup$
    See here for the result about groups of order $equiv2pmod4$.
    $endgroup$
    – Jyrki Lahtonen
    Dec 30 '18 at 14:23
















$begingroup$
See here for the result about groups of order $equiv2pmod4$.
$endgroup$
– Jyrki Lahtonen
Dec 30 '18 at 14:23




$begingroup$
See here for the result about groups of order $equiv2pmod4$.
$endgroup$
– Jyrki Lahtonen
Dec 30 '18 at 14:23


















draft saved

draft discarded




















































Thanks for contributing an answer to Mathematics Stack Exchange!


  • Please be sure to answer the question. Provide details and share your research!

But avoid



  • Asking for help, clarification, or responding to other answers.

  • Making statements based on opinion; back them up with references or personal experience.


Use MathJax to format equations. MathJax reference.


To learn more, see our tips on writing great answers.




draft saved


draft discarded














StackExchange.ready(
function () {
StackExchange.openid.initPostLogin('.new-post-login', 'https%3a%2f%2fmath.stackexchange.com%2fquestions%2f3030334%2fgroup-of-order-6pm-is-solvable-for-prime-p-geq-7%23new-answer', 'question_page');
}
);

Post as a guest















Required, but never shown





















































Required, but never shown














Required, but never shown












Required, but never shown







Required, but never shown

































Required, but never shown














Required, but never shown












Required, but never shown







Required, but never shown







Popular posts from this blog

Quarter-circle Tiles

build a pushdown automaton that recognizes the reverse language of a given pushdown automaton?

Mont Emei